CR: the new Regent microcomputer of high quality

This topic has expert replies
Senior | Next Rank: 100 Posts
Posts: 78
Joined: Wed May 16, 2012 8:57 pm
Thanked: 2 times
According to an independent computer-industry analyst, the new Regent microcomputer is of high quality, is fast, and costs less than any currently existing competing model. It is reasonable to conclude, therefore, as the manufacturer's prospectus does, that the Regent will quickly establish itself as a fast-selling, low-priced alternative to currently available microcomputers.

Which of the following, if true, would LEAST weaken the argument above?

(A) Many retailers already carry one or more low-priced microcomputer models and are disinclined to carry another.
(B) Several faster and lower-priced models of microcomputers will soon be introduced by other computer manufacturers.
(C) The Regent Corporation's microcomputer can be used in conjunction with higher-priced microcomputers manufactured by other companies.
(D) Most of those individuals and companies that could be expected to make up the potential market for the Regent microcomputer have already filled their microcomputer needs.
(E) The independent computer-industry analyst whose assessment was incorporated in the prospectus has used measures of quality that are not universally accepted by the computer-buying public.


I preferred E over C for the following reasons:
E says that the quality measures are NOT universally accepted but author doesn't talk about the sales of Regent's computers in an international market. Maybe, the quality measures are accepted in the potential market of the R computers. So, E isn't a strong -ve or weakener, so, this sounded the best of the lot.

C says the R computers can be used in conjunction with higher-priced computers of other companies (sounds like they can't be used without other company's computers) => their low prices doesn't mean these computer would effectively cost low to its buyer since the buyer would have to buy other company's computers too to make these work as they work in conjunction with others. => it defies "low-priced alternative" => strongly weakens the argument

Experts, please explain in detail where & why my reasons are flawed.

GMAT/MBA Expert

User avatar
Elite Legendary Member
Posts: 10392
Joined: Sun Jun 23, 2013 6:38 pm
Location: Palo Alto, CA
Thanked: 2867 times
Followed by:511 members
GMAT Score:800

by [email protected] » Wed Jun 18, 2014 1:00 am
Hi divineacclivity,

This CR prompt asks us for the answer that would LEAST weaken the argument. This type of question ultimately asks "which 4 answers weaken the prompt and which 1 does NOT." We have to understand the author's logic, then find 4 answers that would weaken it.

The facts:
-An independent computer-industry analyst thinks that the new Regent microcomputer is:
A) High quality
B) Fast
C) Costs less than any currently existing competing model

The conclusion:
-The Regent microcomputer will quickly ESTABLISH itself as a FAST-SELLING, LOW-PRICED alternative to those currently available.

The Logic:
The assessment comes from the ANALYST, so we have to assume that the analyst knows what he/she is talking about. If the computer is everything that the analyst says it is (high quality, fast, low-cost), AND the marketplace is willing to buy it (AND quickly buy lots of them), then the Regent will establish itself as a fast-selling, low-priced alternative.

To weaken the logic....
We have lots of "pieces" that we can attack. For example, we could attack the analyst (maybe his assessment is skewed). We could attack the current state of the marketplace (maybe this computer isn't the cheapest, or maybe it won't be the cheapest for very long, etc.). We can attack the willingness of the customer base to buy the compeer at all (if someone already owns a useful computer, would that person really go out and buy ANOTHER?).

Let's check the answers to see if they weaken/attack any part of the argument:

A: Many retailers are disinclined to carry another low-priced computer. This answer gives a reason WHY the computer might not be available in many locations. It would be tough to establish itself if the customer base can't physically buy it.
B: Several faster/cheaper models will soon be introduced by other companies. These new computers would likely out-compete the Regent in the very aspects that the Regent is attempting to capitalize on (fast and low-cost). This too is a weakener.
C: The Regent can be used with higher-priced computers from other companies. This doesn't seem to attack any of the logical points in the prompt. It works with other computers from other companies? That's NOT a weakener.
D: Most individual/company buyers already have micro-computers. This tells us that there will be far fewer customers than expected.
E: The analyst uses measures that are not accepted by the computer-buying public. The analyst's assessment is PIVOTAL to the prompt. This tells us that the analyst's evaluation won't be one that computer buyer's would agree with. If the buyers don't agree with what the analyst has to say, then THAT could weaken the logic.

The one answer that doesn't attack any part of the prompt is C.

GMAT assassins aren't born, they're made,
Rich
Contact Rich at [email protected]
Image

Senior | Next Rank: 100 Posts
Posts: 78
Joined: Wed May 16, 2012 8:57 pm
Thanked: 2 times

by divineacclivity » Wed Jun 18, 2014 3:44 am
Thank you Rich for your reply.
I don't understand why C is not a weakener, especially because of the clause "in conjunction with other high-priced computers".

My thought process:
C says the R-computers can be used in conjunction with higher-priced computers of other companies (sounds like R-computers can't be used without other company's computers) => their low pricing doesn't benefit its buyer save money since the buyer would have to buy other company's computers too to make these R-computers work since they work (only) in conjunction with others. => it defies/weakens that R-computers are a "low-priced alternative" => strongly weakens the argument

thanks.
~ Divine Acclivity
[email protected] wrote:Hi divineacclivity,

This CR prompt asks us for the answer that would LEAST weaken the argument. This type of question ultimately asks "which 4 answers weaken the prompt and which 1 does NOT." We have to understand the author's logic, then find 4 answers that would weaken it.

The facts:
-An independent computer-industry analyst thinks that the new Regent microcomputer is:
A) High quality
B) Fast
C) Costs less than any currently existing competing model

The conclusion:
-The Regent microcomputer will quickly ESTABLISH itself as a FAST-SELLING, LOW-PRICED alternative to those currently available.

The Logic:
The assessment comes from the ANALYST, so we have to assume that the analyst knows what he/she is talking about. If the computer is everything that the analyst says it is (high quality, fast, low-cost), AND the marketplace is willing to buy it (AND quickly buy lots of them), then the Regent will establish itself as a fast-selling, low-priced alternative.

To weaken the logic....
We have lots of "pieces" that we can attack. For example, we could attack the analyst (maybe his assessment is skewed). We could attack the current state of the marketplace (maybe this computer isn't the cheapest, or maybe it won't be the cheapest for very long, etc.). We can attack the willingness of the customer base to buy the compeer at all (if someone already owns a useful computer, would that person really go out and buy ANOTHER?).

Let's check the answers to see if they weaken/attack any part of the argument:

A: Many retailers are disinclined to carry another low-priced computer. This answer gives a reason WHY the computer might not be available in many locations. It would be tough to establish itself if the customer base can't physically buy it.
B: Several faster/cheaper models will soon be introduced by other companies. These new computers would likely out-compete the Regent in the very aspects that the Regent is attempting to capitalize on (fast and low-cost). This too is a weakener.
C: The Regent can be used with higher-priced computers from other companies. This doesn't seem to attack any of the logical points in the prompt. It works with other computers from other companies? That's NOT a weakener.
D: Most individual/company buyers already have micro-computers. This tells us that there will be far fewer customers than expected.
E: The analyst uses measures that are not accepted by the computer-buying public. The analyst's assessment is PIVOTAL to the prompt. This tells us that the analyst's evaluation won't be one that computer buyer's would agree with. If the buyers don't agree with what the analyst has to say, then THAT could weaken the logic.

The one answer that doesn't attack any part of the prompt is C.

GMAT assassins aren't born, they're made,
Rich

GMAT/MBA Expert

User avatar
Elite Legendary Member
Posts: 10392
Joined: Sun Jun 23, 2013 6:38 pm
Location: Palo Alto, CA
Thanked: 2867 times
Followed by:511 members
GMAT Score:800

by [email protected] » Wed Jun 18, 2014 7:45 pm
Hi Divine Acclivity,

I think I see the issue; you're interpreting the words "can be" to mean "must be."

I read answer C to mean "The new R-Computer can be used with other computers", meaning that the R-Computer is compatible with other computers. This information doesn't weaken the logic in the prompt.

IF answer C had read "The R-computer MUST BE used in conjunction with other higher-priced computers", then I would agree with your point (the answer could be considered a "weakener" because purchasing the R-computer might include some additional expenses). However, that language isn't there.

GMAT assassins aren't born, they're made,
Rich
Contact Rich at [email protected]
Image

Senior | Next Rank: 100 Posts
Posts: 78
Joined: Wed May 16, 2012 8:57 pm
Thanked: 2 times

by divineacclivity » Thu Jun 19, 2014 2:01 am
[email protected] wrote:Hi Divine Acclivity,

I think I see the issue; you're interpreting the words "can be" to mean "must be."

I read answer C to mean "The new R-Computer can be used with other computers", meaning that the R-Computer is compatible with other computers. This information doesn't weaken the logic in the prompt.

IF answer C had read "The R-computer MUST BE used in conjunction with other higher-priced computers", then I would agree with your point (the answer could be considered a "weakener" because purchasing the R-computer might include some additional expenses). However, that language isn't there.

GMAT assassins aren't born, they're made,
Rich
Ok ok. Got it. Thank you very much, Rich.

Senior | Next Rank: 100 Posts
Posts: 66
Joined: Fri Jun 06, 2014 11:48 pm
Followed by:1 members

by phanikpk » Thu Jun 19, 2014 6:15 am
In E, the analyst measures of quality are not universally accepted by computer-buying public, this shows that the prediction might go false, which is a weakener.
C is not a weakener